Difference between revisions of "2011 USAJMO Problems/Problem 3"
(Created page with '== Problem == For a point <math>P = (a, a^2)</math> in the coordinate plane, let <math>\ell(P)</math> denote the line passing through <math>P</math> with slope <math>2a</math>. …') |
|||
Line 2: | Line 2: | ||
For a point <math>P = (a, a^2)</math> in the coordinate plane, let <math>\ell(P)</math> denote the line passing through <math>P</math> with slope <math>2a</math>. Consider the set of triangles with vertices of the form <math>P_1 = (a_1, a_1^2)</math>, <math>P_2 = (a_2, a_2^2)</math>, <math>P_3 = (a_3, a_3^2)</math>, such that the intersections of the lines <math>\ell(P_1)</math>, <math>\ell(P_2)</math>, <math>\ell(P_3)</math> form an equilateral triangle <math>\Delta</math>. Find the locus of the center of <math>\Delta</math> as <math>P_1P_2P_3</math> ranges over all such triangles. | For a point <math>P = (a, a^2)</math> in the coordinate plane, let <math>\ell(P)</math> denote the line passing through <math>P</math> with slope <math>2a</math>. Consider the set of triangles with vertices of the form <math>P_1 = (a_1, a_1^2)</math>, <math>P_2 = (a_2, a_2^2)</math>, <math>P_3 = (a_3, a_3^2)</math>, such that the intersections of the lines <math>\ell(P_1)</math>, <math>\ell(P_2)</math>, <math>\ell(P_3)</math> form an equilateral triangle <math>\Delta</math>. Find the locus of the center of <math>\Delta</math> as <math>P_1P_2P_3</math> ranges over all such triangles. | ||
+ | |||
+ | ==Solution== | ||
+ | |||
+ | Note that all the points <math>P=(a,a^2)</math> belong to the parabola <math>y=x^2</math> which we will denote <math>p</math>. This parabola has a focus <math>F=\left(0,\frac{1}{4}\right)</math> and directrix <math>y=-\frac{1}{4}x</math> which we will denote <math>d</math>. We will prove that the desired locus is <math>d</math>. | ||
+ | |||
+ | First note that for any point <math>P</math> on <math>p</math>, the line <math>\ell(P)</math> is the tangent line to <math>p</math> at <math>P</math>. This is because <math>\ell(P)</math> contains <math>P</math> and because <math>d/dx x^2=2x</math>. If you don't like calculus, you can also verify that <math>\ell(P)</math> has equation <math>y=2a(x-a)+a^2</math> and does not intersect <math>y=x^2</math> at any point besides <math>P</math>. Now for any point <math>P</math> on <math>p</math> let <math>P'</math> be the foot of the perpendicular from <math>P</math> onto <math>d</math>. Then by the definition of parabolas, <math>PP'=PF</math>. Let <math>q</math> be the perpendicular bisector of <math>\oveline{P'F}</math>. Since <math>PP'=PF</math>, <math>q</math> passes through <math>P</math>. Suppose <math>K</math> is any other point on <math>q</math> and let <math>K'</math> be the foot of the perpendicular from <math>K</math> to <math>d</math>. Then in right <math>\Delta KK'P'</math>, <math>KK'</math> is a leg and so <math>KK'<KP'=KF</math>. Therefore <math>K</math> cannot be on <math>p</math>. This implies that <math>q</math> is exactly the tangent line to <math>p</math> at <math>P</math>, that is <math>q=\ell(P)</math>. So we have proved Lemma 1: If <math>P</math> is a point on <math>p</math> then <math>\ell(P)</math> is the perpendicular bisector of <math>\overline{P'F}</math>. | ||
+ | |||
+ | We need another lemma before we proceed. Lemma 2: If <math>F</math> is on the circumcircle of <math>\Delta XYZ</math> with orthocenter <math>H</math>, then the reflections of <math>F</math> across <math>\overleftrightarrow{XY}</math>, <math>\overleftrightarrow{XZ}</math>, and <math>\overleftrightarrow{YZ}</math> are collinear with <math>H</math>. | ||
+ | |||
+ | Proof of Lemma 2: Say the reflections of <math>F</math> and <math>H</math> across <math>\overleftrightarrow{YZ}</math> are <math>C'</math> and <math>J</math>, and the reflections of <math>F</math> and <math>H</math> across <math>\overleftrightarrow{XY}</math> are <math>A'</math> and <math>I</math>. Then we angle chase <math>\angle JYZ=\angle HYZ=\angle HXZ=\angle JXZ=m(JZ)/2</math> where <math>m(JZ)</math> is the measure of minor arc <math>JZ</math> on the circumcircle of <math>\Delta XYZ</math>. This implies that <math>J</math> is on the circumcircle of <math>\Delta XYZ</math>, and similarly <math>I</math> is on the circumcircle of <math>\Delta XYZ</math>. Therefore <math>\angle C'HJ=\angle FJH=m(XF)/2</math>, and <math>\angle A'HX=\angle FIX=m(FX)/2</math>. So <math>\angle C'HJ = \angle A'HX</math>. Since <math>J</math>, <math>H</math>, and <math>X</math> are collinear it follows that <math>C'</math>, <math>H</math> and <math>A'</math> are collinear. Similarly, the reflection of <math>F</math> over <math>\overleftrightarrow{XZ}</math> also lies on this line, and so the claim is proved. | ||
+ | |||
+ | Now suppose <math>A</math>, <math>B</math>, and <math>C</math> are three points of <math>p</math> and let <math>\ell(A)\cap\ell(B)=X</math>, <math>\ell(A)\cap\ell(C)=Y</math>, and <math>\ell(B)\cap\ell(C)=Z</math>. Also let <math>A''</math>, <math>B''</math>, and <math>C''</math> be the midpoints of <math>\overline{A'F}</math>, <math>\overline{B'F}</math>, and <math>\overline{C'F}</math> respectively. Then since <math>\overleftrightarrow{A''B''}\parallel \overline{A'B'}=d</math> and <math>\overleftrightarrow{B''C''}\parallel \overline{B'C'}=d</math>, it follows that <math>A''</math>, <math>B''</math>, and <math>C''</math> are collinear. By Lemma 1, we know that <math>A''</math>, <math>B''</math>, and <math>C''</math> are the feet of the altitudes from <math>F</math> to <math>\overline{XY}</math>, <math>\overline{XZ}</math>, and <math>\overline{YZ}</math>. Therefore by the Simson Line Theorem, <math>F</math> is on the circumcircle of <math>\Delta XYZ</math>. If <math>H</math> is the orthocenter of <math>\Delta XYZ</math>, then by Lemma 2, it follows that <math>H</math> is on <math>\overleftrightarrow{A'C'}=d</math>. It follows that the locus described in the problem is a subset of <math>d</math>. | ||
+ | |||
+ | Since we claim that the locus described in the problem is <math>d</math>, we still need to show that for any choice of <math>H</math> on <math>d</math> there exists an equilateral triangle with center <math>H</math> such that the lines containing the sides of the triangle are tangent to <math>p</math>. So suppose <math>H</math> is any point on <math>d</math> and let the circle centered at <math>H</math> through <math>F</math> be <math>O</math>. Then suppose <math>A</math> is one of the intersections of <math>d</math> with <math>O</math>. Let <math>\angle HFA=3\theta</math>, and construct the ray through <math>F</math> on the same halfplane of <math>\overleftrightarrow{HF}</math> as <math>A</math> that makes an angle of <math>2\theta</math> with <math>\overleftrightarrow{HF}</math>. Say this ray intersects <math>O</math> in a point <math>B</math> besides <math>F</math>, and let <math>q</math> be the perpendicular bisector of <math>\overline{HB}</math>. Since <math>\angle HFB=2\theta</math> and <math>\angle HFA=3\theta</math>, we have <math>\angle BFA=\theta</math>. By the inscribed angles theorem, it follows that <math>\angle AHB=2\theta</math>. Also since <math>HF</math> and <math>HB</math> are both radii, <math>\Delta HFB</math> is isosceles and <math>\angle HBF=\angle HFB=2\theta</math>. Let <math>P_1'</math> be the reflection of <math>F</math> across <math>q</math>. Then <math>2\theta=\angle FBH=\angle C'HB</math>, and so <math>\angle C'HB=\angle AHB</math>. It follows that <math>P_1'</math> is on <math>\overleftrightarrow{AH}=d</math>, which means <math>q</math> is the perpendicular bisector of <math>\overline{FP_1'}</math>. | ||
+ | |||
+ | Let <math>q</math> intersect <math>O</math> in points <math>Y</math> and <math>Z</math> and let <math>X</math> be the point diametrically opposite to <math>B</math> on <math>O</math>. Also let <math>\overline{HB}</math> intersect <math>q</math> at <math>M</math>. Then <math>HM=HB/2=HZ/2</math>. Therefore <math>\Delta HMZ</math> is a <math>30-60-90</math> right triangle and so <math>\angle ZHB=60^{\circ}</math>. So <math>\angle ZHY=120^{\circ}</math> and by the inscribed angles theorem, <math>\angle ZXY=60^{\circ}</math>. Since <math>ZX=ZY</math> it follows that <math>\Delta ZXY</math> is and equilateral triangle with center <math>H</math>. | ||
+ | |||
+ | By Lemma 2, it follows that the reflections of <math>F</math> across <math>\overleftrightarrow{XY}</math> and <math>\overleftrightarrow{XZ}</math>, call them <math>P_2'</math> and <math>P_3'</math>, lie on <math>d</math>. Let the intersection of <math>\overleftrightarrow{YZ}</math> and the perpendicular to <math>d</math> through <math>P_1'</math> be <math>P_1</math>, the intersection of <math>\overleftrightarriw{XY}</math> and the perpendicular to <math>d</math> through <math>P_2'</math> be <math>P_2</math>, and the intersection of <math>\overleftrightarrow{XZ}</math> and the perpendicular to <math>d</math> through <math>P_3'</math> be <math>P_3</math>. Then by the definitions of <math>P_1'</math>, <math>P_2'</math>, and <math>P_3'</math> it follows that <math>FP_i=P_iP_i'</math> for <math>i=1,2,3</math> and so <math>P_1</math>, <math>P_2</math>, and <math>P_3</math> are on <math>p</math>. By lemma 1, <math>\ell(P_1)=\overleftrightarrow{YZ}</math>, <math>\ell(P_2)=\overleftrightarrow{XY}</math>, and <math>\ell(P_3)=\overleftrightarrow{XZ}</math>. Therefore the intersections of <math>\ell(P_1)</math>, <math>\ell(P_2)</math>, and <math>\ell(P_3)</math> form an equilateral triangle with center <math>H</math>, which finishes the proof. | ||
+ | --Killbilledtoucan |
Revision as of 17:29, 30 April 2011
Problem
For a point in the coordinate plane, let denote the line passing through with slope . Consider the set of triangles with vertices of the form , , , such that the intersections of the lines , , form an equilateral triangle . Find the locus of the center of as ranges over all such triangles.
Solution
Note that all the points belong to the parabola which we will denote . This parabola has a focus and directrix which we will denote . We will prove that the desired locus is .
First note that for any point on , the line is the tangent line to at . This is because contains and because . If you don't like calculus, you can also verify that has equation and does not intersect at any point besides . Now for any point on let be the foot of the perpendicular from onto . Then by the definition of parabolas, . Let be the perpendicular bisector of $\oveline{P'F}$ (Error compiling LaTeX. Unknown error_msg). Since , passes through . Suppose is any other point on and let be the foot of the perpendicular from to . Then in right , is a leg and so . Therefore cannot be on . This implies that is exactly the tangent line to at , that is . So we have proved Lemma 1: If is a point on then is the perpendicular bisector of .
We need another lemma before we proceed. Lemma 2: If is on the circumcircle of with orthocenter , then the reflections of across , , and are collinear with .
Proof of Lemma 2: Say the reflections of and across are and , and the reflections of and across are and . Then we angle chase where is the measure of minor arc on the circumcircle of . This implies that is on the circumcircle of , and similarly is on the circumcircle of . Therefore , and . So . Since , , and are collinear it follows that , and are collinear. Similarly, the reflection of over also lies on this line, and so the claim is proved.
Now suppose , , and are three points of and let , , and . Also let , , and be the midpoints of , , and respectively. Then since and , it follows that , , and are collinear. By Lemma 1, we know that , , and are the feet of the altitudes from to , , and . Therefore by the Simson Line Theorem, is on the circumcircle of . If is the orthocenter of , then by Lemma 2, it follows that is on . It follows that the locus described in the problem is a subset of .
Since we claim that the locus described in the problem is , we still need to show that for any choice of on there exists an equilateral triangle with center such that the lines containing the sides of the triangle are tangent to . So suppose is any point on and let the circle centered at through be . Then suppose is one of the intersections of with . Let , and construct the ray through on the same halfplane of as that makes an angle of with . Say this ray intersects in a point besides , and let be the perpendicular bisector of . Since and , we have . By the inscribed angles theorem, it follows that . Also since and are both radii, is isosceles and . Let be the reflection of across . Then , and so . It follows that is on , which means is the perpendicular bisector of .
Let intersect in points and and let be the point diametrically opposite to on . Also let intersect at . Then . Therefore is a right triangle and so . So and by the inscribed angles theorem, . Since it follows that is and equilateral triangle with center .
By Lemma 2, it follows that the reflections of across and , call them and , lie on . Let the intersection of and the perpendicular to through be , the intersection of $\overleftrightarriw{XY}$ (Error compiling LaTeX. Unknown error_msg) and the perpendicular to through be , and the intersection of and the perpendicular to through be . Then by the definitions of , , and it follows that for and so , , and are on . By lemma 1, , , and . Therefore the intersections of , , and form an equilateral triangle with center , which finishes the proof. --Killbilledtoucan